menu
QAmmunity.org
Recent activity by Bertrand Marron
Login
Register
My account
Edit my Profile
Private messages
My favorites
Register
Recent activity by Bertrand Marron
Ask a Question
Questions
Unanswered
Tags
Categories
Ask a Question
Recent activity by Bertrand Marron
Filter
User Bertrand Marron
Recent activity
All questions
All answers
Martha borrows $1200 to buy a desk. She will pay off the loan by paying 2.3% simple interest for 4 years. Martha incorrectly calculates the amount she will pay back using the expression below: 1200(1.023
answered
Dec 21, 2024
Mathematics
high-school
1
answer
42.6k
views
Statistically speaking, who has the highest rates of victimization? A) Older children and boys B) Younger children and boys C) Older children and girls D) Younger children and girls
asked
Nov 23, 2024
Social Studies
high-school
1
answer
202k
views
What is the purpose, vision, and mission of the Texas Reading Academies?
answered
Nov 16, 2024
Social Studies
high-school
1
answer
169k
views
Please help me and show work
asked
Nov 13, 2024
Mathematics
college
2
answers
111k
views
________ is best described as a set of goal-directed actions a firm takes to gain and sustain superior performance relative to competitors. A) Credo B) Competency management C) Strategy D) Behavior modification
asked
Oct 4, 2024
Business
high-school
1
answer
88.8k
views
Kim is watching a rocket launch from an observation spot 6 miles away. Find the angle of elevation from Kim to the rocket, which is at a height of 5.7 miles. a) 38.1 degrees b) 45 degrees c) 53.1 degrees
answered
Sep 28, 2024
Mathematics
high-school
1
answer
135k
views
Question 10 of 20: Select the best answer for the question. 10. What are the two factors that can cause the volume of water in the ocean to change?
answered
Aug 10, 2024
Geography
college
1
answer
141k
views
Climatological Seasons: (Also know these for the NH only.)
answered
Jul 29, 2024
Geography
high-school
1
answer
4.7k
views
A nurse is caring for a client who is experiencing preeclampsia and has a new prescription for IV magnesium sulfate. Which of the following medications should the nurse anticipate administering if the
answered
Jul 8, 2024
Medicine
college
1
answer
111k
views
US Mexican war summary
asked
Jun 13, 2024
History
high-school
1
answer
113k
views
A line has a slope of ā 2/5 and passes through the point (17, ā 6). write its equation in slope-intercept form.
asked
Apr 10, 2024
Mathematics
high-school
1
answer
19.5k
views
(15m + 20p) + 25q = 15m + (20p + 25q) What property is this? Identity Property Distributive Property Associative Property Commutative Property
answered
Mar 10, 2024
Mathematics
middle-school
2
answers
43.8k
views
The points obtained by students of a class in a test are normally distributed with a mean of 60 points and a standard deviation of 5 points. About what percent of students have scored between 60 and 65
asked
Feb 7, 2024
Mathematics
high-school
1
answer
198k
views
The plan of care for a client with osteoporosis includes active and passive exercises, calcium supplements, and daily vitamins. How does a nurse determine that the desired effect of therapy has been achieved?
asked
Jan 27, 2024
Medicine
college
1
answer
97.7k
views
A 32-year-old woman presents with dysuria, back pain, and fever. Which test or tests are required for making an appropriate diagnosis in this patient? a) Urinalysis b) Urinalysis and contrast-enhanced
answered
Jan 25, 2024
Health
high-school
1
answer
199k
views
Read the excerpt from "To the Public" by William Lloyd Garrison. Assenting to the "self-evident truth" maintained in the American Declaration of Independence, "that all men are created equal, and endowed
answered
Dec 11, 2023
History
college
2
answers
59.7k
views
CAN SOMEONEPLEASE HELP iām struggling so much on this lollll
answered
Nov 3, 2023
Mathematics
high-school
1
answer
157k
views
a restaurant used 8 1 /6 pounds of rice on Monday and 5 3/4 pounds of rice on Tuesday. How many more pounds of rice did the restaurant use on Monday than Tuesday
answered
Sep 18, 2023
Mathematics
high-school
1
answer
59.6k
views
The bear population increases at a rate of 2% per year. If there are 1,200 bears currently, how many bears will there be in 10 years?
asked
Sep 11, 2023
Mathematics
high-school
1
answer
69.8k
views
Imagenes de feliz dia de san valentin para mi familia.
asked
Aug 7, 2023
Spanish
high-school
1
answer
219k
views
Ask a Question
Welcome to QAmmunity.org, where you can ask questions and receive answers from other members of our community.
9.3m
questions
12.0m
answers
Search QAmmunity.org